Sei sulla pagina 1di 41

-BLAKE-

Introduction to communications systems

Example 1.1 Introduction to communications systems

Calculate the wavelength in free space corresponding to


a frequency of:
Example1.4
(a) 1 MHz (AM radio broadcast band)
(b) 27 MHz (CB radio band) A receiver has a noise power bandwidth of 10
(c) 4 GHz (used for satellite television) kHz. A resistor that matches the receiver input
impedance is connected across its antenna terminals.
Solution
What is the noise power contributed by that resistor in
the receiver bandwidth, if the resistor has temperature
C= λ
of 27 ⁰C

Solution
λ= T(K)= T (⁰C) + 273
= 27 + 273
= 300 K

PN = kTB
(a)
= (1.38 X 10-23 J/K (300 K) (10 X 103 Hz)
= 4.14 X 10-17 W
= 300m

EXAMPLE 1.5
(b)
A 300Ω resistor is connected across the 300 Ω
= 11.1m antenna input of a television receiver. The bandwidth of
the receiver is 6 MHz, and the resistor is at room
temperature (293 K or 20⁰C or 68⁰F). Find the noise
(c) power voltage applied to the receiver input.

= 0.075m Solution
= 7.5cm
PN = kTB
= (1.38 X 10-23 J/K (300 K)(293 K) (6 X 103 Hz)
= 24.2 X 10-15 W
= 24.2 fW
V N = √4kTBR
= √4 (1.38 X 10-23 J/K)(293)(6 X 106 Hz) (300 Ω)
= 5.4 X 10-6 V
= 5.4µV
noise power is 30mW. What is the amplifier noise figure,
as a ratio?

Solution:
Introduction to communications systems 100µW
100
(S/N)i = = 100
EXAPMLE 1.6
A diode noise generator is required to produce NF (ratio) = =3
1µW
10µV of noise in a receiver with an input impedance of
33.5
75 Ω, resistive, and a noise power bandwidth 0f 200
kHz. (These values are typical of FM broadcast 1µW
receivers.) What must be current through the diode be? (S/N)o = = 33.3
0.03W
Solution
VN
IN =
I N = √2qI0B
R
2 Introduction to communications systems
I N = 2qI0B
10µV EXAMPLE 1.10
I
2
N The signal at the input of an amplifier has an
= S/N of 42dB. If the amplifier has noise figure of 6dB,
= what is the S/N at the output (in decibel)?
75 Ω
2qB Solution
= 0.133µA NF (dB) = (S/N)i(dB) - (S/N)o(dB)
(S/N)o(dB) = (S/N)i(dB) - NF (dB)
(0.133 X 10 -6 A)2
= 42 dB – 6 dB
= 36dB
=
EXAMPLE 1.11
An amplifier has a noise figure of 2 dB. What is
2(1.6 X 10 -19 C)(200 X 103 Hz)
its equivalent noise temperature?
Solution
= 0.276 A or 276 mA NF (dB) = 10 log NF (ratio)
EXAMPLE 1.8 NF(dB)
A receiver produces a noise power of 200 mW NF (ratio)= antilog
with no signal. The output level increases to 5W when a 10
signal is applied. Calculate (S + N)/N as a power ratio = antilog 0.2
and in decibels. = 1.585
Teq = 290 (NF – 1)
Solution: = 290 (1.585 – 1)
5W = 169.6 K
(S+N)/N=
0.2 W
= 25
(S+N)/N (dB) = 10 log 25
= 14dB
EXAMPLE 1.9
The signal power at the input to an amplifier is
100µW. at the output, the signal power is 1W and the
0 =

√1+2V =
4π √LC 2

1
2
C0
0 =

1+2V =
2 2
4π 0 L

Radio-Frequency Circuit 1
(C0/ C) 2-1
EXAMPLE 2.5 =
A varactor has a maximum capacitance of 80pF V=
and is used in a tuned circuit with a 100 µH inductor. 4π (2 X 1..78 X 106) 2 (100 X 10-6)
2

2
(a) Find the resonant frequency with no tuning = 20 X 10-12 F
voltage applied (80/20) 2-1
(b) Find the tuning necessary for the circuit to = 20pF
resonate at double the frequency found in part =
(a)
2
Solution
(a) = 7.5 V
1
0 =

2π√LC
1
=
2π√(100 X 10-6)(80 X 10-12) Radio-Frequency Circuit
= 1.78 MHz
EXAMPLE 2.6
(b) A portable radio transmitter has to operate at
1 temperature form – 50⁰C to 350⁰C if its signal is derived
C0 from a crystal oscillator with a temperature coefficient
= of +1 ppm/degree C and it transmits at exactly 146 MHz
0
at 20⁰C, find the transmitting frequency at the two
C= extremes of the operating temperature range.
2π√LC
√1+2V Solution
T = 0 +k 0 (T – T0)

1
C0
max
-6
= 8 MHz
=146 MHz + (146 MHz)(1 X 10 )(35

-20)
= 146.00219 MHz
-6
min
=146 MHz + (146 MHz)(1 X 10 )(-5

-20)
= 145.99635 MHz Radio-Frequency Circuit

EXAMPLE 2.9
EXAMPLE 2.7 Configure a simple PLL synthesizer using a 10
Sine – wave signals with frequencies of 10 MHz MHz crystal so that it will generate the AM broadcast
frequencies from 540 to 1700 kHz.
11MHz are applied to square – law mixer. What
Solution
frequencies appear at the output?
OSC

Q=
Solution
ref

1 = 11MHz 2 2 = 22MHz 10 MHz


=
+ =21 MHz 10 kHz
1 2
= 1000
2 = 11MHz 2 2 = 20MHz
0 =N ref

1 - 2 = 1 MHz
0

N =
EXAMPLE 2.8 ref
A phase – locked loop has a VCO with a free –
running frequency of 12 MHz. As the frequency of the
540 kHz
reference input is gradually raised from zero, the loop N =
locks at 10MHz and comes out of lock again at 16 MHz. 10 kHz
= 54
(a) Find the capture range and lock range
(b) Suppose that the experiment is repeated, but 1700 kHz
this time the reference input begins with a very N =
high frequency and steadily moves downward. 10 kHz
Predict the frequencies at which lock would be = 170
achieved and lost.

Solution
(a)
(b)
Capture range = 2(12 MHz – 10 MHz)
12 MHz + 2 MHz =14 MHz
= 4 MHz
12 MHz – 4 MHz = 8 MHz
lock range = 2(16 MHz – 12 MHz)
100 – 10
1.8 MHz
= = 20 kHz
90

Radio-Frequency Circuit
EXAMPLE 2.10
The synthesizer in Figure 2.40 has P = 10 and ref

=10 kHz. Find the minimum frequency step size and Amplitude Modulation
compare it with that obtained using a fixed divided-by-
10 prescaler. EXAMPLE 3.1
A carrier wave with an RMS voltage of 2V and a
Solution frequency of 1.5 MHz is modulated by a sine wave with
Step size = M ref
a frequency of 500 Hz and amplitude of 1V RMS. Write
the equation for the resulting signal.
= 10 X 10 kHz
= 10 kHz Solution

0 = (M+NP) ref Ec = √2 X 2V
= (M+NP)10 kHz = 2.83 V

’0 = (M+1+NP) ref
Em = √2 X 1V
= 1.41 V
= (M+1+NP)10 kHz
ωc = √2π X 1.5 X 106
’ 0 + 0 = (M+1+NP) 10 kHz – (M+NP)10kHz = 9.42 X 106 rad/s
= (M+1+NP – M - NP) 10 kHz
= 10kHz ωm = √2π X 500
= 3.14 X 103 rad/s

EXAMPLE 2.11 V(t) = (Ec+Em sin ωm t) sin ωc t


A synthesizer of the type shown in figure 2.42 = [2.83 + 1.41 sin (3.14 X 103t)] sin (9.42 X
106 t)V
has ref =20 kHz and local oscillator operating at 10 Mhz.

Find the frequency range of the output as the value of N EXAMPLE 3.2
ranges from 10 to 100. Also find the minimum amount Calculate m for the signal in Example 3.1, and
by which the frequency can be varied. write the equation for this signal in the form of Equation
(3.5).
Solution
Solution
0 = N 0+ LO
Em
0 = 10 X 20 kHz + 10 MHz m=
Ec
= 10.2 MHz
1
0 = 100 X 20 kHz + 10 MHz =
2
= 12 MHz
= 0.5
12 MHz – 10.2 MHz
V(t) = Ec (1+ m sin ωmt) sin ωct
Step size =
= 2.83 [1 + 0.5 sin (3.14 X 103t)] sin (9.42 X
106 t)

Amplitude Modulation
Amplitude Modulation

EXAMPLE 3.3 EXAMPLE 3.7


Find the modulation index if a 10V carrier is
amplitude modulated by three different frequencies An AM broadcast transmitter has a carrier
with amplitudes of 1V, 2V, and 3V, respectively. power output of 50kW. What total power would be
produced with 80% modulation?
Solution
1 2 Solution
3 P t=Pc 1 + m2
M1= = 0.1 M2 = = 2
0.2 M2 = = 0.3
= (50kW) 1 + 0.82
10 10
10 2
= 66kW

M1 = √m2 + m2 + m2
EXAMPLE 3.10
= √0.1 + 0.2 + 0.3
1 2
2
3 2 2

= 0.374 An transmitter generates an LSB signal with a


carrier frequency of 8 MHz. What frequency will appear
at the output with a two tone modulating signal with
frequencies of 2 kHz and 3.5 kHz?
EXAMPLE 3.6
Solution
CB radio channels are 10kHz apart. What is the
8 MHz – 2 KHz = 7.998 MHz
maximum modulation frequency that can be used if a
And
signal is to remain entirely within its assigned channel? 8 MHz – 3.5 kHz = 7.9965 MHz

Solution
B = 2Fm
Or
B
Fm =
2
10 kHz
=
2
= 5 kHz
Angle Modulation
Angle Modulation
EXAMPLE 4.3
EXAMPLE 4.1 An FM broadcast transmitter operates at its
An FM modulator has kƒ = 30 kHz/V and maximum deviation of 75kHz. Find the modulation
operates at a carrier frequency of 175 MHz. Find the index for sinusoidal modulating signal with a frequency
output frequency for an instantaneous value of the of:
modulating signal equal to:
(a) 15 kHz (b) 50 kHz
(a) 150 mV (b) -2V
Solution
Solution (a)
δ
6 3
(a) ƒ sig = (175 X 10 Hz) + (30 X 10 Hz/V)(150 X mƒ =
10-3 V) ƒm
= 175.0045 X 106 Hz
= 175.0045 MHZ 75 kHz
(b) ƒ sig = (175 X 106 Hz) + (30 X 103 Hz/V)(-2V) =
= (175 X 106 Hz) - (30 X 103 Hz/V)(2V) 15 kHz
= 174.94 X 106 Hz
= 174.94 MHZ = 5.00
(b)
EXAMPLE 4.2 δ
The same Fm modulator as in the previous mƒ =
example is modulated by a 3V sine wave.Calculate the ƒm
deviation.
75 X 103 kHz
=
Solution
50 kHz
Em = 3 √2V
= 4.24 V = 1500

δ = kƒEm
= 30 kHz/V X 4.24 V
= 127.2 kHz

Angle Modulation
EXAMPLE 4.4
A phase modulator has kp = 2 rad/V. what RMS Solution
voltage of a sine wave would cause a peak phase δ
deviation of 60⁰? mƒ =
ƒm
Solution φ = mf
360⁰ = 2π rad
2π rad X 60 δ
60⁰ = =
360⁰ ƒm
π
= rad 5000
3 =
300
φ = 16.7 rad
kp =
ℓm EXAMPLE 4.6
A phase modulator has a sensitivity of kp = 3
φ rad / V. How much frequency deviation does it produce
ℓm = with a sine – wave input of 2V peak at frequency of
kp 1kHz?
(π/3) rad
= Solution
2 rad/V φ = kpEmsin ωmt

π mp = φmax
= V = k pEm
6 = 3 rad/V X 2V
= 0.524 V = 6 rad

Vpeak δ
VRMS = mƒ =
√2V ƒm

0.524 δ = mffm
= = 6 X 1 kHz
= 6 kHz
√2V
= 0.37 V

Angle Modulation

EXAMPLE 4.9
Angle Modulation An FM signal has frequency deviation of 5 kHz
and a modulating frequency of 1 kHz. The signal – to –
EXAMPLE 4.5
noise ratio at the input to the receiver detector is 20dB.
An FM communication transmitter has
Calculate the approximate signal – to noise ratio at the
maximum frequency deviation of 5 kHz and range of
detector output.
modulating frequencies from 300 Hz to 3 kHz. What is
the maximum phase shift that it produces? Solution
(S/N)(dB) A transmitter has a carrier power of 10 W at an
-1
ES/EN = log efficiency of 70 %. How much power must be supplied
20 by the modulating amplifier for 100 % modulation?
20
= log -1 Solution
20 P0
= 10 η=
EN/Es =1/10 Ps
= 0.1
P0
φ ≈ EN/Es Ps =
= 0.1 rad η

mfN = 0.1 10
=
δN = mffm 0.7
= 0.1 X 1 kHz
= 100 Hz = 14.3 W

(ES/EN) 0 = δs/δN Pa = 0.5 Ps


= 5 kHz / 100 Hz = 7.14 W
= 50

(S/N)0 (dB) = 20 log 50


= 34dB

Transmitters

EXAMPLE 5.3
Transmitters A transmitter operates from 12 V supply, with a
collector current of 2A. The modulation transformer has
EXAMPLE 5.1
A crystal oscillator is accurate within 0.0005%. a turns ratio of 4 : 1. What is the load impedance seen
by audio amplifier?
How far off frequency could its output be at 27 MHz?
Solution
Solution
0.0005 Vcc
Za =
27 X 106 Hz X = 135 Hz
100 Ic
12 V
EXAMPLE 5.2 =
2A
=6Ω
A PLL FM generator has the block diagram
2
N1 shown in Figure 5.25, with ƒref = 100kHz, N= 200, and kf =
Zp = Za 50 kHz/V.
N2
(a) Calculate the carrier frequency of the output
= (6 Ω)(42)
signal
= 96 Ω
(b) What RMS modulating voltage will be required
EXAMPLE 5.4 for a deviation of 10 kHz at the carrier
A collector-modulated Class C amplifier has a frequency?
carrier output power Pc of 100 W and an efficiency of
(a) fc =Nfref
70%. Calculate the supply power and the transistor
Vp
power dissipation with 100% modulation
= 200 X 100 kHz
Solution VRMS =
= 20 MHz
P0 =1.5 Pc
= 1.5 X 100 W √2
= 150 W
(b) δ
P0 0.2 V
Ps = kf =
η =
Vp
150 W
= √2
0.7
= 214 W Δ
PD = P S – P 0 = 0.141 V
= 214 W – 150 W Vp =
= 64 W = 141 mV
kf

10 kHz
=
EXAMPLE 5.5 50 kHz/V
An AM transmitter is required to produce 10 W = 0.2 V
of carrier power when operating from a 15 V supply.
What is the required load impedance as seen from the
collector?

Solution
V2cc
RL =
2Pc
152
= Receivers
2 X 10
= 11.25 Ω EXAMPLE 6.1
A tuned circuit tunes the AM radio broadcast
band (from 540 to 1700 kHz). If its bandwidth is 10 kHz
EXAMPLE 5.21 at 540 kHz, what is it at 1700 kHz?
Solution Solution
1700
B = 10 kHz X 540 (a) Fimage = fsig +2f IF
= 17.7 kHz = 590 kHz + 2(455 kHz)
= 1500 kHz
EXAMPLE 6.4 (b) fimage fsig
A receiver has a sensitivity of 0.5 µV and a X= _
blocking dynamic range of 70 dB. What is the strongest fsig fimage
signal that can be present along with a 0.5 µV signal
1500 kHz 590 kHz
without blocking taking place?
= _
590 kHz 1500 kHz
Solution
= 2.149
P1 V1
Asig
(dB) = 20 log
P2 V2 = √1+Q 2
X2
Aimage
P1
(dB) = 70
P2 √
= 1 + 402 X 2.1492
= 85.97
V1 P1/P2 (dB) IR (dB) = 20 log 85.97
= antilog = 38.7 dB
V2 20
P1/P2 (dB)
V1 = V2 antilog
20
70
= (0.5 µV) antilog
20
= 1581 µV
= 1.58 mV

Receiver

EXAMPLE 6.7
An FM detector produces a peak – to – peak
Receiver output voltage of 1.2 V from an fm signal that is
modulated to 10 kHz deviation by sine wave. What is
EXAMPLE 6.5 the detector sensitivity?
The receiver in figure 6.5 is tuned to a station at
590 kHz. Solution
(a) Find the image frequency V0 peak – to – peak
(b) Calculate the image rejection in decibels, V0 peak =
assuming that the input filter consists of one 2
tuned circuit with a Q of 40. 1.2 V
=
2
= 0.6 V
√Q Q p s

1
0.6 V =
kd = √40 X 30
10 kHz = 0.0289
= 60 µV/Hz
(b) kopt = 1.5 kc
EXAMPLE 6.8 = 1.5 X 0.0289
A PLL FM detector uses a VCO with kf= 100 = 0.0433
kHz/V. If it receives an FM signal with a deviation of 75
kHz and sine – wave modulation, what is the RMS (c) B = kf0
output voltage from the detector? = 0.0433 X 455 kHz
= 19.7 kHz
Solution
δ
V0 peak =
kf

75 kHz
=
100 kHz/V
= 0.75 V

V0 peak
V0 RMS =
√2
0.75 V
= = 0.53 V
√2

Receiver

EXAMPLE 6.11
An S-meter of the type described above reads S-
Receiver
6. Calculate the signal strength at the receiver input.
EXAMPLE 6.9
An IF transformer operates at 455 kHz. The Solution
V1
primary circuit has a Q of 40 and the secondary has a Q
dB = 20 log V2
of 30. Find V1 = 50 µV
(a) The critical coupling V1 dB
(b) The optimum coupling factor = antilog
(c) The bandwidth using the optimum coupling V2 20
factor
V1
Solution V2 =
Antilog (dB/20)
(a) 1
kc = 50 µ
= sampling rate of 44.1 kHz. What audible frequency
Antilog (18/20) would result?
= 6.29 µV
Solution
fa = fs – fm
= 44.1 kHz – 30 kHz
= 14.1 kHz

EXAMPLE 7.3
Calculate the number of levels if the number of
bits per sample is?
(a) 8 (as in telephony)
(b) 16 (as in compact disc audio systems)

Solution
(a) N = 2m
= 28
= 256

(b) N = 2m
= 216
= 65,536

Digital Communications
Digital Communications
Example 7.1
A telephone line has a bandwidth of 3.2 kHz and EXAMPLE 7.4
a signal – to – noise ratio of 35 dB. A signal is Find the maximum dynamic range for a linear
transmitted down this line using a four – level code. PCM system using 16 – bit quantizing.
What is the maximum theoretical date rate?
Solution
Solution DR = 1.76 + 6.02m dB
C = 2B log2 M = 1.76 + 6.02 X 16
= 2 (3.2 X 103) X log2 4 = 98.08 dB
= 12.8 X 103 b/s
= 12.8 kb/s
EXAMPLE 7.5
S/N = antilog10 (35/10) Calculate the minimum data rate needed to
= 3162 transmit audio with sampling rate of 40 kHz and 14 bits
per sample
C = B og2 (1 + S/N)
= (3.2 X 103) log2 (1 + 3162) Solution
= 37.2 kb/s D = fsm
= 40 X 103 X 14
EXAMPLE 7.2 = 560 X 103 b/s
An attempt is made to transmit a baseband = 560 kb/s
frequency of 30 kHz using a digital audio system with a
EXAMPLE 7.6 EXAMPLE 8.5
A signal at the input to a mu – law compressor is Find the (suppressed) carrier frequency for
positive, with its voltage one – half the maximum value. channel 5 of a group.
What proportion of the maximum output voltage is Solution
produced? Fc = 64 kHz + 4 (12 – 5) kHz = 92 kHz

Solution
V0 ln(1 + µvi/Vi) EXAMPLE 8.6
v0 = A 2kHz tone is present on channel 5 group 3 of a
ln (1 + µ) supergroup. At what frequency does the tone appear in
the supergroup output?
V0 ln(1+255X0.5)
= Solution
ln (1 + 255) Fg = 92 kHz – 2 kHz = 90 kHz

= 0.876 V0 Fsg = 516 kHz – 90 kHz = 426 kHz

Data Transmission
The Telephone System
EXAMPLE 9.2
EXAMPLE 8.1 Calculate the maximum efficiency of an
A local loop has a resistance of 1 kΩ, and the asynchronous communication system using ASCII with
telephone connected to it has an off – hook resistance seven data bits, one start bit, one stop bit, and one
of 200 Ω. Calculate the loop current and the voltage parity bit.
across the telephone when the phone is:
(a) On hook Solution
(b) Off hook η = N D / NT
= 7 / 10
Solution = 0.7 or 70 %
RT = 1000 Ω + 200 Ω = 1200 Ω
EXAMPLE 9.4
I = 48 V/ 1200 Ω = 40 mA Generate the vertical and longitudinal
redundancy checks, using odd parity, for the block
V = IR = 40 mA X 200 Ω = 8 V STX L R C ETB

EXAMPLE 8.3 Solution


A telephone signal takes 2 ms to reach its Bits STX L R C
destination. Calculate the via net loss required for an ETB
acceptable amount of echo (MSB) 7 0 1 1 1
0
Solution 6 0 0 0 0
VNL = 0.2t + 0.4 dB 0
= 0.2 X 2 + 0.4 dB
= 0.8 dB
5 0 0 0 0 0 0 1 1 1
1 0 0
4 0 1 0 0 0 0 0 0 0
0 1 1
3 0 1 0 0 0 0 0 0 0
1 0 1
2 1 0 1 1 0 0 0 1 1
1 1 0
1 0 0 0 1 1 1 0 0 0
1 1 0
0 0 1 1 0
Bit STX L R C ETB 1 0
Bit STX L R C ETB | LRC 0 1 0 0 0
7 0 1 1 1 0 1 1
7 0 1 1 1 0 | 0 Solution
6 0 0 0 0 0
6 0 0 0 0 0 | 1 0 1 1 1 1
5 0 0 1 0 1 0 1
5 0 0 1 0 1 | 1 0 0 1 1 1
4 0 1 0 0 0 0 0
4 0 1 0 0 0 | 0 0 0 0 0 0
3 0 1 0 1 1 1 1 – Error in this row
3 0 1 0 0 1 | 1 0 0 0 0 0
2 1 0 1 1 1 0 1
2 1 0 1 1 1 | 1 0 0 0 1 1
1 0 0 0 1 1 1 0
1 0 0 0 1 1 | 1 1 1 0 0 0
__________________________________________ 1 0
_____________________________ 0 0 1 1 0
VRC 0 0 0 0 1 1 0
VRC 0 0 0 0 1 | 0 0 1 0 0 0
1 1
|
Error in
this column

0 1 1 1 1 0 1
0 1 1 1 1
0
0 0 1 1 1 0 0
0 0 1 1 1
Data Transmission
0
EXAMPLE 9.5 0 0 0 1 0 1 1
The following block contains exactly one error. 0 0 0 1 0
Find the error and decode the block. Odd parity is used. 1
0 0 0 0 0 0 1
0 1 1 1 1 0 0 0 0 0
0 1 0
0 0 0 1 1 1 0 (a) Using any system?
0 0 0 1 1 (b) Using a code with four possible state?
1 Solution
1 1 0 0 0 1 0 (a) S 15
= log -1
1 1 0 0 0
N 10
1
0 0 1 1 0 1 0 C = B log2 (1 + S/N)
0 0 1 1 0 = 10 X 103 log2 (1+31.6)
1 = 10 X 103 X 5.03
0 1 0 0 0 1 1 = 50.3 kb/s
S B a u d
E C = 2B log2M
= 2 X 10 X 103 X 2
T = 40 kb/s
T

X
B

Digital Modulation and Modern

EXAMPLE 12.2
A modulator transmits symbols, each of which
has sixty – four different possible states, 10,000 times
Data Transmission
per second. Calculate the baud rate bit rate
EXAMPLE 9.6
Solution
How many hamming bits are required for a
C = Slog2M
block length of 21 message bits = 10 X 103 X log264
= 60 kb/s
Solution
2n ≥ m + n + 1 EXAMPLE 12.3
25 ≥ 21 + 5 + 1 The GSM cellular radio system uses GMSK in a
32 ≥ 27
200 – kHz channel, with a channel data rate of 270.833
2n ≥ m + n + 1 kb/s Calculate:
24 ≥ 21 + 4 + 1 (a) The frequency shift between mark and space
16 ≥ 26 (b) The transmitted frequencies if the carrier
(center) frequency is exactly 880 MHz

Solution
(a)
Digital Modulation and Modern fm – fs = 0.5fb = 0.5 X 270.833 kHz = 135.4165
kHz
EXAMPLE 12.1 (b)
A radio channel has a bandwidth of 10 kHz and fmax = fc + 0.25fb = 880 MHz - 0.25 X 270.833
a single – to – noise ratio of 15dB. What is the kHz = 880.0677 kHz
maximum data rate that can be transmitted?
fmax = fc - 0.25fb = 880 MHz - 0.25 X 270.833 1 MHz
kHz = 879.9329 kHz N= 5
180 kHz

EXAMPLE 12.6 (d) C = 2B log2 M


A typical dial – up telephone connection has a C
bandwidth of 3 kHz and a signal – to – noise ratio of 30 B=
2 log2 M
dB. Calculate the Shannon limit.
56 kHz
Solution
=
S/N = antilog (30/10)
2 log2 4
= 1000
= 14 kHz
C = B log2 (1 + S/N)
= 3 X 103 X log2 1001
1 MHz
= 29.9 kb/s
N= = 71
14 kHz

Multiplexing and Multiple – Access Techniques


Multiplexing and Multiple – Access Techniques
Example 13.2
Example 13.1
A voice transmission occupies a channel 30 kHz
How many of each of the following signals
wide. Suppose a spread – spectrum system is used to
would fit into a 1 MHz spectrum allocation?
(a) Voice, with a maximum frequency of 4kHz, increase its bandwidth to 10 MHz. If the signal has a
modulated using SSBSC AM. total signal power of – 110 dBm at the receiver input
(b) The same voice signal using DSB full – carrier and the system noise temperature referred to the same
AM. point is 300 K, calculate the signal – to – noise ratio for
(c) High – fidelity music with a maximum baseband both systems.
frequency of 15 kHz, using wideband FM with a
maximum deviation of 75 kHz. Solution
(d) A bit stream at 56 kb/s, using QPSK modulation
PN(30 kHz) = 1.38 X 10 -23 J/K X 300 K X 30 X 103
and assuming a noise – less channel.
Hz
Solution = 124 X 10 -18 W
(a) 1MHz = -129 dBm
N= = 250
4 kHz S/N = -110 dBm – ( - 129 dBm) = 19 dB

(b) 1MHz S/N = - 110 dBm – (- 104 dBm ) = - 6 dB


N= = 125
8 kHz

(c) B = 2 (δmax + fmmax = 2(75 kHz + 15 kHz ) = 180 EXAMPLE 13.3


kHz
A Frequency – hopping spread – spectrum GP (dB) = (S/N)i(dB) – (S/N)0 (dB)
system hops to each of 100 frequencies every 10 (S/N)0(dB) = (S/N)i(dB) – Gp (dB)
seconds. How long does it spend on each frequency? = 20 dB – 17 dB
= 3 dB
Solution
t = 10 seconds / 100 hops
= 0.1 second per hop

EXAMPLE 13.4
A digital communication scheme uses DQPSK to
transmit a compressed PCM audio signal which has a bit
rate of 16 kb/s. the chipping rate is 10 to 1. Calculate the
number of signal changes (symbols) which must be
transmitted each second.

Solution
160 / 2 = 80 kilobaud

Transmission Lines

EXAMPLE 14.1
A coaxial cable has capacitance of 90 pF/m and
a characteristics impedance of 50 Ω. Find the
Multiplexing and Multiple – Access Techniques inductance of 1m length.
Example 13.5 Solution
A signal would have a bandwidth of 200 kHz and L
a signal – to – noise ratio of 20 dB if transmitted without Z0 =
spreading. It is spread using a chipping rate of 50:1. C
Calculate its bandwidth and a signal – to – noise ratio
after spreading. L
2
Z =
Solution 0 C
BRF
GP = L =2Z C
BBB 0
= 502 X 90 X 10 -12 H/m
BRF = GPBBB = 225 nH/m
= 50 X 200 kHz
= 10 MHz EXAMPLE 14.2
Find the characteristic impedance of each of the
following lines:
GP(dB)= 10 log GP (a) An open – wire line with conductors 3 mm in
= 10 log 50 diameter separated by 10 mm
= 17 dB
(b) A coaxial cable using a solid polyethylene
dielectric having Єr = 2.3, with a inner conductor Solution
2 mm in diameter and an outer conductor 8 mm vp =vfc
= 0.66 X 300 X 106 m/s
in inside diameter
= 198 X 106 m/s
Solution
vp = fλ
vp
(a) Z0≈ 276 log D / r
λ =
= 276 log 10 mm /1.5 mm
f
= 227 Ω
198 X 106 m/s
(b) 138 D
=
Z0 = log
200X 106 Hz
√Є r d = 0.99 m

138 8 L = 0.99 m X 45⁰ / 360⁰


= log = 0.124 m
√2.3 2
= 54.8 Ω

Transmission Lines

EXAMPLE 14.3
A 50 Ω line is terminated in a 25Ω resistance.
Transmission Lines Find the SWR.
Solution
EXAMPLE 14.3 Z0
Find the velocity factor and propagation velocity SWR =
for a cable with a Teflon dielectric (Єr = 2.1 ). ZL

Solution 50
1 =
vf = 25
=2
√Є r

1 EXAMPLE 14.7
= A generator sends 50mW down a 50Ω line. The
√2.1 generator is matched to the line, but the load in not. If
= 0.69 the coefficient of reflection is 0.5, how much power is
reflected and how is dissipated in the load?
vp
vf = Solution
c Pr = T2Pi
= 0.52 X 50 mW
vp= vf c = 12.5 mW
= 0.69 X 300 X 106 m/s
PL = P i – P r
EXAMPLE 14.5 = 50 mW – 12.5 mW
What length of standard RG – 8/U coaxial cable = 37.5 mW
would be required to obtain a 45 phase shift at 200
MHz? PL = Pi (1 – T2)
= 50 mW X (1 – 0.52) =50 Ω
= 37.5 mW 50 Ω +j(100 Ω) tan45⁰

EXAMPLE 14.8 100 Ω + j(50 Ω)


A transmitter supplies 50 W to a load through a = 50 Ω
line with an SWR of 2:1. Find the power absorbed by the 50 Ω + j (100 Ω)
load.
100 Ω + j(50 Ω)
Solution =
4SWR 1 + j2
PL = Pi
(1 + SWR)2 [100 Ω] + j (50 Ω)](1 – j2)
=
4X2 (1 + j2)(1 – j2)
= X 50 W
2
(1 + 2)
= 44.4 W 100 Ω + j (50 Ω) – j(200 Ω) +100

=
1+4
Transmission Lines
200 Ω - j(150 Ω)
EXAMPLE 14.9 =
Calculate the impedance looking into a 50 Ω line 5
1m long, terminated in load impedance of 100 Ω, if the = 40 Ω - j(30 Ω)
line has a velocity factor of 0.8 and operates at a
EXAMPLE 14.10
frequency of 30 MHz A series tuned circuit operating at a frequency
of 1 GHz is to be constructed from a shorted section of
Solution
V air – dielectric coaxial cable. What length should be
λ= used?
f
Solution
vfc vp = vfc
= = 0.95 X 300 X 106 m/s
f = 285 X 106 m/s

0.8 X 300 X 106 m/s vp = fλ


=
30 X 106 Hz vp
=8m fλ=
f
1m
θ= X 360⁰ 285 X 106 m/s
8m =
= 45⁰ 1000 X 106 Hz

ZL + jZ0 tanθ = 0.285 m


Z = Z0
Z0 +jZL tanθ λ
L=
100 Ω + j( 50 Ω) tan 45⁰ 2
Solution
0.285 m v pt
= 0.8(300 X 106 m/s)(1.4 X 10-6 s)
2 d=
= 0.143 m =
2
2

vfct
= 168 m
=
2
Transmission Lines

EXAMPLE 14.11
A transmitter is required to deliver 100W to an Transmission Lines
antenna through 45 m coaxial cable with a loss of 4
EXAMPLE 14.19
dB/100 m. What must be the output power of the Two adjacent minima on a slotted line are 23 cm
transmitter, assuming the line is matched? apart. Find the wavelength and the frequency, assuming
a velocity factor.
Solution
4dB
Solution
loss (dB) = 45 m X
λ = 2 X 23 cm
100 m
= 46 cm
= 1.8 dB
vp = fλ
Pin 1.8
= antilog
vp
Pout 10
f =
= 1.51
λ
Pin = 1.51 X 100 W
vfc
= 151 W
=
EXAMPLE 14.12
λ
Normalize and plot impedance of 100 + j25Ω on
50-Ω line. 0.95 X 300 X 106 m/s
=
Solution 0.46 m
Z = 620 X 106 Hz
z= = 620 MHz
Z0
EXAMPLE 14.20
100 + j25 Ω The forward power in a transmission line is
=
150W, and the reverse power is 20W. Calculate the SWR
25 Ω
= 2 + j0.5 on the line.
EXAMPLE 14.18
A TDR display shows a discontinuity 1.4 µs from Solution
Pr 20W
the start. If the line has a velocity factor of 0.8, how far
=
is the fault from the reflectometer? Pi 150W
= 0.365 PD =
4πr2
1 + 0.365
SWR = 100 W
1 – 0.365 =
4π (10 X 103 m)2
= 2.15 = 79.6 nW/m2

Radio – Wave Propagation


Radio – Wave Propagation
EXAMPLE 15.4
EXAMPLE 15.1 Find the electric field strength for the signal in
Find the characteristic impedance of the previous example
polyethylene, which has a dielectric constant of 2.3
Solution
Solution √30Pt
377
E =
L = r
√Є r

√30 X 100
377 =
= 10 X 103
√2.3 = 5.48 mV/m
= 249 Ω
EXAMPLE 15.5
EXAMPLE 15.2 A transmitter has a power output of 150 W at a
The dielectric strength of air is about 3MV/m. carrier frequency of 325 MHz. it is connected to an
Arcing is likely to take place at field strengths greater antenna with a gain of 12 dBi. The receiving is 10 km
than that. What is the maximum power density of an away and has a gain of 5 dBi. Calculate the power
electromagnetic wave in air? delivered to the receiver, assuming free – space
propagation. Assume also that there are no losses or
Solution mismatches in the system.
E
PD =
Solution
L
Lfs = 32.44 + [20 log d (km)] + [20 log f
(3 X 106)2 (MHz)] – [GT(dBi)] – [GR (dBi)]
= = 32.44 + 20log 10 + 20log 325 – 12
377 –5
= 23.9 GW/m2 = 85.7 dB

EXAMPLE 15.3 PT
A power of 100 W is supplied to an isotropic 10 log = 85.7
radiator. What is the power density at a point 10km PR
away?
PT 85.7
log =
Solution
PR 10
Pt
PT 85.7 (b) d = √17hT + √17hR
= antilog
PR 10 = √17 X 1.5 + √17 X 1.5
= 10.1 km
PT
PR =
antilog(85.7/ 10)

150 W
=
-9
= 404 X 10 W or 404 nW
372 X 106 Radio – Wave Propagation

Radio – Wave Propagation EXAMPLE 15.10


Find the propagation loss for a signal at 800
MHz, with a transmitting antenna height of 30m, over a
distance of 10 km, using:
EXAMPLE 15.8 (a) The free – space model (Equation 15.32)
(b) The mobile – propagation model (Equation
The critical frequency at a particular time is 11.6 15.33)
MHz. What is the MUF for transmitting station if the
required angle of incidence for propagation to a Solution
destination is 70⁰? (a) LFS = 32.44 + 20 log d + 20 log f
= 32.44 + 20 log 10 + 20 log 800
Solution = 110.5 dB
fc
fm = (b) Lp = 68.75 + 26.16 log f – 13.82 log h + (44.9
cos θ1 – 6.55 log h) log d
= 68.75 + 26.16 log 800 – 13.82 30 + (44.9
11.6 – 6.55 log 30) log 10
= = 159.5 dB
Cos 70⁰
= 33.9 MHz EXAMPLE 15.11
An automobile travels at 60 km/hr. Find the
time between fades if the car uses:
(a) A cell phone at 800 MHz
(b) A PCS phone at 1900 MHz

EXAMPLE 15.9 Solution


A taxi company uses a central dispatcher, with 60 km 60 X 103 km
an antenna at the top of a 15m tower, to communicate =
with taxicabs. The taxi antennas are on the roofs of the hr 3.6 X 103 s
cars, approximately 1.5 m above the ground. Calculate = 16.7 m/s
the maximum communication distance:
(a) Between the dispatcher and a taxi (a) C
(b) Between two taxis T=
2fv
Solution
300 X 106
(a) d = √17hT + √17hR =
= √17 X 15 + √17 X 1.5 2 X 800 X 106 X 16.7
= 21.0 km = 11.2 ms
η =
(b) C RT
T= 67
2fv =
67 + 5
300 X 106 = 0.93 or 93 %
=
2 X 1900 X 106 X 16.7 Antennas
= 4.7 ms
EXAMPLE 16 .3
Radio – Wave Propagation Two antennas have gains of 5.3 dBi and 4.5 dBd,
respectively, Which has greater gain?
EXAMPLE 15.12
A metropolitan area of 1000 square km is to be Solution
covered by cells with a radius of 2 km. how many cell G = 4.5 dBd
sites would be required, assuming hexagonal cells? = 4.5 + 2.14 dBi
= 6.64 dBi
Solution
A EXAMPLE 16.4
N= A dipole antenna has an efficiency of 85%.
3.464r2 Calculate its gain in decibels.

1000 Solution
N= 2.14
3.464 X 22 D = log-1
= 73 10
= 1.638

G = Dη
Antennas = 1.638 X 0.85
= 1.39
EXAMPLE 16 .1
Calculate the length of a half – wave dipole for G(dBi) = 10 log 1.39
an operating frequency of 20 MHz. = 1.43 dBi

Solution EXAMPLE 16.6


142.5 The ERP of a transmitting station is specified as
L = 17 W in a given direction. Express this as an EIRP in dBm
f so that it can be used with the path loss equations in
Chapter 15.
142.5
= Solution
20 ERP
= 7.13 m ERP (dBm) = 10 log
1mW
EXAMPLE 16.2 = 10 log (17 X 103)
A dipole antenna has a radiation resistance of = 42.3 dBm
67Ω and a loss resistance of 5 Ω, measured at the
feedpoint. Calculate the efficiency. EIRP(dBm) = ERP(dBm) + 2.14 dB
= 42.3 + 2.14
Solution = 44.44 dBm
Rr
Antennas
EXAMPLE 16 .7

Antennas

EXAMPLE 16 .7 (b)
A helical antenna with eight turns is to be 15NS(πD)2
constructed for frequency of 1.2 GHz G =
(a) Calculate the optimum diameter and spacing for λ
the antenna and find the total length of the
15 X 8 X 0.0625 (π X 0.08) 2
antenna
=
(b) Calculate the antenna gain in dBi
0.253
(c) Calculate the bandwidth.
= 30.3
Solution
= 14.8 dBi
(a) c
λ =
(c)
f
52 λ λ
θ =
300 X 106
πD NS
=
1200 X 106
52 X 0.25 0.25
= 0.25 m
=
π X 0.08 8 X 0.0625
λ
D =
= 36.6⁰
π

0.25
=
π
= 0.08 m
= 80 m

λ
S =
4

0.25
=
4
= .0625 m
= 62.5 m

L = NS
Antennas
= 8 X 62.5 mm
EXAMPLE 16 .8
= 500 mm
Design a log – periodic antenna to cover the
frequency range from 100 to 300 MHz. Use τ = 0.7 and α
= 30⁰
Solution

142.5
L1
L =
L2 = Microwave Devices
f
τ
EXAMPLE 17 .1
Find the cutoff frequency for the TE10 mode in
142.5
an air – dielectric waveguide with an inside cross section
0.445
L = of 2cm by 4cm. Over what frequency range is the
= dominant mode the only one that will propagate?
90
Solution
0.7
c
= 1.58 m
f=
= 0.636 m 2a
142.5 300 X 106 m/s
L2 =
L = 2 X 4 10 -2 m
L3 = = 3.75 X 109 Hz
320 = 3.75 GHz
τ
= 0.445 m

0.636
L1 α
= EXAMPLE 17.2
= tan Find the group velocity for the waveguide in
0.7 Example 17.1 at a frequency of 5 GHz.
2D1 2
Solution
= 0.909 m 2
vg = c 1-
fc
L1 f 2
D1 =
= (300 X 106 m/s) 1-
2 tanα 3.75
2 = 198 X 106 m/s 5
0.445
=
2 tan 15⁰
= 0.830 m
Microwave Devices EXAMPLE 17.5
Find the characteristic impedance of the
EXAMPLE 17 .3 waveguide used in the previous examples, at a
A waveguide has a cutoff frequency for the frequency of 5 GHz.
dominant mode of 10GHz. Two signals with frequencies
of 12 and 17 GHz propagate down a 50 m length of the Solution
guide. Calculate the group velocity for each and the Z0 = 377 Ω
2
difference in arrival time for two. 1-
fc
f
Solution
vg = c 1- 2 = 377 Ω
fc 1- 2

f 2
3.75
= (300 X 106 m/s) 1- 5
10 = 570 Ω
= 165.8 X 106 m/s 12

50 m
t1 = = 301.6 ns EXAMPLE 17.6
165.8 X 10 m/s 6 Find the guide wavelength for the waveguide
used in the previous examples.
50 m
t2 = = 206.1 ns Solution
242.6 X 10 m/s 6 vp
λg =
t1 - t2 = 301.6 ns – 206.1 ns =95.5 ns f

EXAMPLE 17.4 454 X 106 m/s


Find the phase velocity for the waveguide used =
in Example 17.1 and 17.2, at a frequency of 5 GHz. 5 X 106 Hz
= 0.0908 m
= 9.08 cm
Solution
vp = c
2
1-
fc
f

vp = 300 X 106 m/s


2
1-
3.75
5
= 4.54 X 108 m/s

Microwave Devices

Microwave Devices EXAMPLE 17.7


A signal with a level of 20 dBm enters the main A pulsed magnetron operates with an average
waveguide of a directional coupler in the direction of power of 1.2kW and a peak power of 18.5 kW. One
the arrow. The coupler has an insertion loss of 1dB, pulse is generated every 10 ms. Find the duty cycle and
coupling of 20 dB, and directivity of 40 dB. Find the the length of a pulse.
strength of the signal emerging from each guide. Also,
find the strength of the signal that would emerge from Solution
Pavg = PPD
the secondary guide if the signal in the main guide were
Pavg
propagating in the other direction. D =
PP
Solution
20 dBm – 1dB = 19 dBm 1.2
=
20 dBm – 20dB = 0 dBm 18.5
= 0.065 or 6.5%
0 dBm – 40 dB = -40 dBm
Ton
EXAMPLE 17.8 D =
A Gunn device has a thickness of 7 µm. At what TT
frequency will it oscillate in the transit – time mode? Ton = D TT
= 0.065 X 10 ms
Solution = 0.65 ms
d
t= EXAMPLE 17.10
v A pyramidal horn has an aperture (opening) of
58 mm in the E plane and 78 mm in the H plane. It
7 X 10-6 m
operates at 10 GHz. Calculate:
=
(a) Its gain in dBi
1 X 105 m/s
(b) The beamwidth in the H plane
(c) The beamwidth in the E plane
= 7 X 10 -11 s
Solution
1
(a)
f=
c
T
λ =
f
1
=
300 X 106
7 X 10 -11 s
=
10 X 106
= 14.3 X 109 Hz
= 0.03 m
= 14.3 GHz

Microwave Devices
Microwave Devices
EXAMPLE 17.10

7.5 dEdH
EXAMPLE 17.9
G =
λ2 300 X 106
vp =
300 X 106 2 X 109 √2
=
= 0.106 m
10 X 106
= 0.03 m
EXAMPLE 17.12
(b)
A radar transmitter as a power of 10kW and
70 λ
θH = operates at a frequency of 9.5 GHz. Its signal reflects
dH from a target 15 km away with a radar cross section of
10.2 m2. The gain of the antenna is 20 dBi. Calculate the
70 X 0.03 received signal power.
=
0.078 Solution
= 26.9⁰ c
λ2PTG2σ
(c) λ =
56 λ PR =
θE = f
dH
(4π)3r4
56 X 0.03
300 X 106
=
0.058 0.03162(10 X 102)(1002)(10.2)
= 29 ⁰ vp =
=
9.5 X 106
(4π) 3 (15 X 103) 4
= 0.0316 m
= 10.1 X 10 -15 W

dB
= 10.1 fW
G = antilog
10

20
= antilog
10
= 100
Microwave Devices

EXAMPLE 17.11
Calculate the approximate dimension for a
square patch antenna for a frequency of 2 GHz, on a
substrate with a relative permittivity of 2
Microwave Devices
Solution
c EXAMPLE 17.13
vp = A pulse sent to target returns after 15 µs. How
f √Єr far away is the target?

Solution
96 km/h= m/s
ct 3600
R= = 26.7 m/s
2
2vrfi
(300 X 106)(15 X 10-6) fD = m/s
= c
2 2( 26.7) (10 X 109)
= 2250 m =
= 2.25 km 300 X 106
= 1.778 kHz
EXAMPLE 17.14
A pulse radar emits pulses with a duration of
1µs and a repetition rate of 1 kHz. Find the maximum
and minimum range for this radar.

Solution

c
Rmax =
2f

300 X 106
=
2 X 1000
= 150 X 103 m
= 150 km

cTP
Rmax =
2

(300 X 106) (1 X 10 -6)


=
2
= 150 m

Terrestrial Microwave Communication Systems

EXAMPLE 18.1
Suppose that the transmitter and receiver
EXAMPLE 17.15 towers have equal height. How high would they have to
Find the Doppler shift caused by a vehicle be to communicate over a distance of 40 km?
moving toward a radar at 60 mph, if the radar operates
at 10 GHz. Solution

Solution d= √17 h + √17 h


T R

60 mph = 60 X 1.6 km/h


d =2 √17 hT
= 96 km/h

96 X 1000 d2
h=
68 = 20 + 25 – (32.44 + 20 log 40 + 20
log 6000)
402 = -95 dB
= 2W
68 PT (dBm)= 10 log = 33 dBm
= 23.5 m 1mW
PR (dBm) = 33 dBm – 95 dB = -62 dBm
EXAMPLE 18.2
A line – of – sight radio link operating at a EXAMPLE 18.4
frequency of 6 GHz has a separation of 40 km between In a microwave system, the antenna sees a sky
antennas. An obstacle in the path is located 10km from temperature of 120 K, and the antenna feedline has a
the transmitting antenna. By how much must be beam loss of 2dB. Calculate the noise temperature of the
clear the obstacle? antenna/feedline system, referenced to the receiver
input.
Solution
d1d2 Solution
R = 10.4 L = antilog (2/10) = 1.58
f (d1 + d2 )
(L – 1) 290 + Tsky
Ta=
10 X 30 L
R = 10.4
6 (10 + 30 ) (1.58 – 1) 290 + 120
=
= 11.6 m 1.58
= 182 K

EXAMPLE 18.5
A receiver has a noise figure of 2dB. Calculate its
equivalent noise temperatue.

Solution
NF = antilog (2/10) = 1.58

Teq = 290 (NF – 1)


Terrestrial Microwave Communication Systems = 290 (1.58 – 1)
= 168 K
EXAMPLE 18.3
A Transmitter and a receiver operating at 6 GHz
are separated by 40 km. How much power (in dBm) is Terrestrial Microwave Communication Systems
delivered to the receiver if the transmitter has an
output power of 2W, the transmitting antenna has a EXAMPLE 18.6
gain of 20 dBi, and the receiving antenna has a gain of The antenna and feedline combination from
25 dBi? example 18.4 is used with the receiver from example
18.5. Calculate the thermal noise power in dBm,
Solution referred to the receiver input, if the receiver has a
PR bandwidth of 20 MHz.
(dB) = GT(dBi) + GR(dBi) – (32.44 + 20 log
d + 20 log f) Solution
PT TN(system) = Ta + Teq
= 182 K + 168 K
= 350 K
Calculate the local oscillator frequency if the receiver
PN = kTB uses low – side injection.
= 1.38 X 10 -23 J/K X 350 K X
20 MHz Solution
= 96.6 fW
FLO = 6870 MHz – 70 MHz = 6800 MHz
PN(dBm) = 10 log (96.6 fW/1 mW)
= - 100 dBm EXAMPLE 18.10
A microwave repeater has the block diagram
shown in Figure 18.9(a). The received signal has a
EXAMPLE 18.7 carrier frequency of 6870 MHz, and the transmitted
Calculate the carrier – to – noise ratio, for the
signal is to have a carrier frequency of 6710 MHz. the IF
signal in Example 18.3, received by the installation in
is 70 MHz. What should be the frequencies of the local
Example 18.6
oscillator and the shift oscillator? To what frequency
should the output of mixer #3 be tuned? Verify these
Solution
PR = - 62 dBm results by following the signal through the repeater.

PN = -100 dBm Solution


Fso = 6870 MHz – 6710 MHz = 160 MHz
C/N= -62 – ( - 100) = 38 dB
FLO = Fo – 70 MHz
EXAMPLE 18.8 = 6710 MHz – 70 MHz
The system in Example 18.7 operates at a bit of = 6640 MHz
40 Mb/s. Calculate the energy per bit to noise density
6640 MHz + 160 MHz = 6800 MHz
ratio, in decibels.
6870 MHz – 6800 MHz = 70 MHz
Solution
fo = 70 MHz + 6640 MHz = 6710
PR = antilog (-62/10) mW
MHz
N0 = kT
= 631 pW
= 1.38 X 10 -23 X 350

= 4.83 X 10 -21 W/Hz


PR
Eb 15.8 X 10 -18
Eb =
= 10 log = 35.1
dB
f
N0 4.83 X 10 -21

631 pW
=
40 Mb/s
= 15.8 X 10 -18 J
Terrestrial Microwave Communication Systems
Television
EXAMPLE 18.9
In the receiver in Figure 18.7(b), the received
Example 19.1
carrier frequency is 6870 MHz and the IF is 70 MHz.
A video signal has 50% of the maximum Solution
luminance level. Find its level in IRE units. Y = 0.30R + 0.59G + 0.11B
= (0.30 X 0.2) + (0.59 X 0.4) + (0.11 X
Solution 0.8)
IRE = 7.5 + 0.5 X 92.5 = 0.384
= 53.75 IRE units
I = 0.60R – 0.28G – 0.32B
EXAMPLE 19.2 = (0.62 X 0.2) – (0.28 X 0.4) – (0.32 X
Calculate the total percentage of the signal time 0.8)
that is occupied by: = - 0.248
(a) Horizontal blanking
(b) Vertical blanking Q = 0.21R – 0.52G + 0.31B
(c) Active video = (0.21 X 0.2) – (0.52 X 0.4) + (0.31 X
0.8)
Solution = 0.082
(a)
10 EXAMPLE 19.5
horizontal blanking (%) = X 100 What proportion of the maximum transmitter
63.5 power is used to transmit a black setup level?
Solution
= 15.7 % P (black setup) = 0.6752
= 0.456 or 45.6%
(b) 42 EXAMPLE 19.6
vertical blanking (%) = X 100 Suppose a television receiver tuned to channel
252
6 radiates a local oscillator signal into the cable. What
= 8.0 %
channel will be interfered with?
(c)
Solution
fLO = 83.25 + 45.75
active video (%) = (100 – 15.7) X 0.92
= 77.6 % = 129 MHz

EXAMPLE 19.3 EXAMPLE 19.7


Consider a digital video signal that has a
A typical low – cost monochrome receiver has a
video bandwidth of 3 MHz. What is horizontal resolution of 640 by 480 pixels, with a 30 Hz frame rate
and progressive scan. The luminance is sampled using 8
resolution in line?
bits per sample. The two chroma channels also use 8
Solution bits per sample, but the color resolution in one – fourth
LH = B (MHz) X 80 that used for luminance. Find the approximate bit rate
= 3 X 40 for this signal, neglecting synchronization, error
= 240 lines correction and compression.

Solution
NPL = NVNH
= 480 X 640 = 307.2 X 10 3 pixels

Television NPT= 1.5 NPL = 460.8 X 103 pixels

Example 19.4 fb = NPTmRf


An RGB video signal has normalized values of R = 460.8 X 103 X 8 X 30
= 0.2, G = 0.4, B =0.8. Find the values of Y, I, and Q = 110.6 Mb/s
Satellite Communication
Solution
EXAMPLE 20.1 (b) 4 X 1011
Find the velocity and the orbital period of a v=
satellite in a circular orbit (d + 6400)
(a) 500 km above the earth’s surface
(b) 36,000 km above the earth’s surface
( approximately the height of geosynchronous 4 X 1011
=
satellite)
(36,000 + 6400)
Solution
= 3.07 km/s
(a) 4 X 1011
v=
r = 6400 km + 36,000 km = 42.4 Mm
(d + 6400)
C = 2πr
= 2π X 42.4 km
4 X 1011
= 266.4 Mm
=
(500 + 6400)
C
T=
= 7.6 km/s
V
r = 6400 km + 500 km = 6900 km
266.4 X 106 m
=
C = 2πr
3.07 X 103 m/s
= 2π X 6900 km
= 43.4 Mm
= 86.8 X 103 s
= 24 hours
C
T=
V

43.4 X 106 m
=
7.6 X 103 m/s

= 5.71 X 103 s
= 1.6 hours

Satellite Communication

EXAMPLE 20.2
Calculate the angle of declination for an
antenna using a polar mount at a latitude of 45⁰.
Solution
Satellite Communication
6400sin45⁰
EXAMPLE 20.1 θ = arctan
36 X 103 + 6400 ( 1 - 45⁰
)
feedhorn, is 0.4 dB, and the LNA has a noise
temperature of 40 K. Calculate G/T.
= 6.81⁰
Solution
EXAMPLE 20.3 G = 40 dBi – 0.4 dB
Calculate the length of the path to = 39.6 dBi
geostationary satellite from an earth station where the
angle of elevation is 30⁰. L = antilog (0.4/10) = 1.096

Solution ( L – 1) 290 + Tsky


Ta =
d = √ (r + h)2 – ( rcos θ)2 – r sin θ
L
d = √ (6400 + 36 X 103)2 – ( 6400 cos
30⁰)2 – 6400 sin 30⁰ ( 1.096 – 1) 290 + 15
= 39 X 103 km Ta =
1.096
EXAMPLE 20.4 = 39 K
A satellite transmitter operates at 4 GHz with a
transmitter power of 7 W and an antenna gain of 40dBi. G/T (dB) = GR (dBi) – 10 log (Ta + Teq)
= 39.6 – 10 log (39 +40)
The receiver has an antenna gain of 30dBi, and the path
= 20.6 dB
length is 40,000 km. Calculate the signal strength at the
receiver.

Solution
PR EXAMPLE 20.6
(dB) = GT (dBi) + GR (dBi) – (32.44 + A receiver has noise figure 1.5 dB. Find its
20 log d + 20 log f) equivalent noise temperature
PT
= 40 + 30 – (32.44 + 20 log (40 Solution
+ 103) + 20 log 4000) NF = antilog ( 1.5/10)
= - 126.5 dB = 1.41

7W Teq = 290 (NF – 1)


PT(dBm) = 10 log = - 88 dBm = 290 (1.41 – 1)
1 mW = 119 K

PR(dBm) = 38.5 dBm – 126.5 dB = -88


dBm

Satellite Communication
Satellite Communication
EXAMPLE 20.7
EXAMPLE 20.5 The receiving installation whose G/T was found
A receiving antenna with a gain of 40 dBi looks in Example 20.5 is used as a ground terminal to receive
at a sky with a noise temperature of 15 K. The loss a signal from a satellite at a distance of 38,000 km. The
between the antenna and the LNA input, due to the satellite has a transmitter power of 50 watts and
antenna gain of 30 dBi. Assume losses between the
satellite transmitter and its antenna negligible. The A Vehicle travels through a cellular system at
frequency is 12 GHz. Calculate the carrier – to – noise 100 kilometer per hour. Approximately how often will
ratio at the receiver, for a bandwidth of 1 MHz. handoffs occur if the cell radius is:
(a) 10 km
Solution (b) 500 m
PT (dBW) = 10 log 50 = 17 dBW
Solution
EIRP (dBW)= 17 dBW + 30 dBi = 47 dBw 100 km/hr X 1000 m/km
v=
FSL (dB) = 32.44 + 20 log d + 20 log f 3600 s/hr
= 32.44 + 20 log 38,000 + 20 = 27.8 m/s
log 12,000
= 205.6 dB d
C/N(dB) = EIRP(dBW) – FSL(dB) – L misc + t=
G/T – K(dBW) – 10 log B v
= 47 dBW – 205.6 dB + 20.6
1 X 103 m
dB + 228.6 dBW – 10log (1 X 106)
=
= 30.6 dB
27.8 m/s
EXAMPLE 20.9
= 36 s
A typical TVRO installation for use with C – band
satellites ( downlink at approximately 4 GHz) has a has a EXAMPLE 21.2
diameter of about 3 cm and an efficiency of about 55%. A cellular telephone system uses a 12 – cell
Calculate its gain and beamwidth. repeating pattern. There are 120 cells in the system and
20,000 subscribers. Each subscriber uses the phone
Solution
average 30 minutes per day, but on average 10 of those
c
λ= minutes are used during the peak hour.
f
(a) The average and peak traffic in erlangs for the
300 X 106 m/s whole system
= (b) The average and peak traffic in erlangs for one
4 X 109 HZ cell, assuming callers are evenly distributed over
= 7.5 cm the system
(c) The approximate average call – blocking
0.55 π2 X 32 probability
λ= (d) The approximate call – blocking probability
0.0752
during the peak hour.
= 8.69 X 103
= 39 dB
Solution
70λ
(a) 0.5
θ=
416
D
70 X 0.075 T = 20,000 X
= t=
3 24
= 1.75⁰ 120
= 416 E
Satellite Communication = 3.47 E

EXAMPLE 21.1 10
3333
T = 20,000 X
t= 30,144
60 =
120 544
= 3333 E = 55,412 batches/min
= 27.8 E
Address/min = batches/min X
address/batch
= 55.412 X 16
Satellite Communication = 886

EXAMPLE 21.3 Paging and Wireless Data Networking


Calculate the maximum distance between base
and mobile that can be accommodated with a guard
time of 123µs.
EXAMPLE 23.2
Solution Calculate the efficiency, in terms of bits per
d = ct
second per hertz of RF bandwidth, for the FLEX system
= 300 X 106 m/s X 123 X 10-6 s
= 36.9 km at its maximum data rate.

Solution
Personal Communication Systems 6400 b/s
= 0.256 b/sHz
EXAMPLE 22.1 25 kHz
A CDMA mobile measures the signal strength
from the base as -100 dBm. What should the mobile
transmitter power be set to as a first approximation?
EXAMPLE 23.3
Solution Calculate the maximum and minimum hopping
PT = -76 dB – PR
rate for the Bluetooth system.
= -76 dB – (-100 dBm)
= 24 dBm
Solution
= 250 mW
1
fh (max) =
Paging and Wireless Data Networking
625 µs
EXAMPLE 23.1
Suppose the POCSAG system is used with simple
= 1600 Hz
tone pagers, which require only an address field. If all
the frames are used for address, how many pages could 1
be transmitted by this system in 1 minute if it operates fh (min) =
at the slowest POCSAG rate of 512b/s? Assume that 5 X 625 µs
only one preamble is needed.
= 320 Hz
Solution
Total number of usable bits/minute =
512 X 60 – 576 = 30, 144 bits

bits/min
batches /min =
bits/batch
Fiber Optics

Fiber Optics
EXAMPLE 24.3
EXAMPLE 24.1 A single – mode fiber has a numerical aperture
A fiber has an index of refraction of 1.6 for the of 0.15. What is the maximum core diameter it could
core and 1.4 for the cladding. Calculate: have for use with infrared light with a wavelength of 820
(a) The critical angle nm?
(b) θ2 for θ1 = 30⁰
(c) θ2 for θ1 = 70⁰ Solution
Solution 0.383λ
(a) rmax =
n2 N.A.
θc = arcsin
n1 0.383 X 820 X 10 -9 m
1.4 rmax =
= arcsin 0.15
1.6 = 2.1 X 10 m -6

= 61⁰ = 2.1 µm
(b)
n1
sin θ2 = sin θ1
n2
EXAMPLE 24.4
1.6 An optical fiber has a bandwidth – distance
=
product of 500 MHz –km. if a bandwidth of 85 MHz is
1.4 sin 30⁰
= 0.571 required for a particular mode of transmission, what is
θ2 = arcsin 0.571 the maximum distance that can be used between
= 34.8⁰ repeaters?

EXAMPLE 24.2 Solution


Calculate the numerical aperture and the Bandwidth X distance = 500 MHz – km
maximum angle of acceptance for the fiber described in 500 MHz – km
distance =
Example 24.1
bandwidth
Solution
500 MHz – km
N.A. = √n - n
1 2
=
85 MHz
= √1.6 – 1.4
2 2
2 2
= 5.88 km
= 0.775

arcsin 0.775 = 50.8⁰


Fiber Optics
Fiber Optics
EXAMPLE 24.7
EXAMPLE 24.5 An optical has one input (port 1) and two
The fiber whose dispersion equation is given in outputs (port 2 and 3). Its specification are given below.
Equation (24.11) has zero dispersion at a wavelength of Even though negative signs are not used for the
1310 nm and has a zero – dispersion slope of 0.05 ps/ coupling specifications, they are implied. Since this is a
(nm2 . km). Calculate the total dispersion of 50 km of passive coupler, the output power must always be less
this fiber when it is used with source having a line width than the input power.
of 2 nm at a wavelength of 1550 nm.
Input port Output port
Solution Coupling dB
S0 λ
4 1 2
Dc(λ) = λ- 0 ps/km (nm . 3
km ) 1 3
4 λ3 6
2 1
40
0.05 13104
2 3
Dc(λ) = 1550 -
4 1550 3 40
3 1
= 9.49 ps/km (nm . km ) 40
D = DcΔλ 3 2
= 9.49 ps/(nm . km) X 2 nm 40
= 18.98 ps/km Find :
(a) The percentage of the input power that
Δt = Dt emerges from each of ports 2 and 3 when the
= 18.98 ps/km input is at port 1
= 949 ps (b) The directivity
EXAMPLE 24.6 (c) The excess loss in decibels
Find the bandwidth and bandwidth – distance
product for the fiber in Example 24.5 Solution
(a) P2 -3
Solution = log -1 = .501 = 50%
1 Pin 10
B=
2Δt
P3 -6
1 = log -1
= .251 = 25.1%
B= Pin 10
2 X 949 ps
= 526.8 MHz
526.8 MHz X 50 km = 26.3 GHz – km
Fiber – Optic System

EXAMPLE 25.1
A fiber – optic link extends for 40 km. The laser
– diode emitter has an output power of 1.5 mW, and the
receiver requires a signal strength of -25 dBm for
Fiber Optics
satisfactory signal – to – noise ratio. The fiber is
EXAMPLE 24.8 available in length of 2.5 km and can be spliced with a
Find the energy, in electron – volts, in one loss of 0.25 dB per splice. The fiber has a loss of 0.3
photon at a wavelength of 1µm. dB/km. The total of all the connector losses at the two
ends is 4dB. Calculate the available system margin.
Solution
C Solution
f= Pin(dBm) = 10 log Pin(mW)
λ = 10 log 1.5
= 1.76 dBm
3 X 108 m/s
f= span length/ fiber length = 40 km / 2.5 km
-6
1 X 10 m = 16
= 3 X 1014 Hz
Connector losses
E = hf 4 dB
= (6.26 X 1034 )(3 X 1014 ) Fiber loss : 40 km X 0.3 dB/km
= 1.99 X 10-19 J 12 dB
Splice loss : 15 splice X 0.25 dB/splice
1 eV = 1.6 X 10-19 J 3.75 dB
Total
1
19.75 dB
1J = eV
1.6 X 10-19
Pout = 1.76 dBm – 19.75 dB
= 6.25 X 1018 eV
= -17.99 dBm
E = (1.99 X 10-19 ) (6.25 X 1018 )
System margin = - 17.99 dBm – (-25 dBm)
= 1.24 eV
= 7.01 dB
EXAMPLE 24.9
A typical photodiode of the type shown in
EXAMPLE 25.2
Figure 24.25 has an input optical power of 500 nW. A 45 km length of fiber must not lengthen
Calculate the diode current. pulses by more than 100 ns. Find the maximum
permissible value for the pulse – spreading constant.
Solution
1 = 500 nW X 0.33 A/W
Solution
= 150 nA
Δt
D=
L

100 ns
D=
45 km 500 500
= 2.22 ns/km D = =
= 1 ns/km
Bl 500

Fiber – Optic System


Fiber – Optic System
EXAMPLE 25.5
EXAMPLE 25.3 A fiber – optic system uses a detector with a rise
Calculate the maximum data rate for the 45 km time of 3ns and a source with a rise time of 2ns. If RZ
fiber system in the previous example when it is used code is used with a data rate of 100Mb/s over a distance
with a transmitter having a rise time of 50 ns and a of 25 km, calculate the maximum acceptable dispersion
receiver having a rise time of 75 ns, if the code is for the fiber and the equivalent bandwidth – distance
(a) NRZ
product.
(b) RZ
Solution
Solution
1
TRT =√T 2 + T2 + T2 fb =
2TRT
= √ 50
Rtx Rrx Rf
2
+ 75
2
+ 100
2
= 135 ns 1
(a) fb =
1 2Tf
fb = 1
TRT =
1 2 X 100 X 106 b/s
= = 5 ns
135 ns
= 7.4 MHz 3.46 ns
(b) D= = 0.1386 ns/km
1 25 km
fb =
2TRT
1 500
= Bl = = 3608 MHz – km =
2 X 135 ns 3.61 GHz -km
= 3.7 MHz 0.1386

EXAMPLE 25.4
A Fiber is rated as having a bandwidth –
distance product of 500 MHz – km. Find its dispersion in
ns/km, and find the rise time of pulse in 5km length of
this cable.

Solution
500
Bl =
D

Potrebbero piacerti anche